You are on page 1of 22

1

Bài viết về bất đẳng thức Schur,


Vornicu−Schur và kỹ thuật pqr

Nguyễn Huy Tùng


Lớp 8A1, THCS Hồng Bàng, Hải Phòng
Email: cr7forward@gmail.com

Trong thế giới bất đẳng thức hiện nay, bất đẳng thức Schur là một bất đẳng thức mạnh. Tuy
nhiên nó không được đưa vào chương trình phổ thông và đây là một sự thiếu sót khá lớn.
Bài viết này sẽ cung cấp cho các bạn một cái nhìn về bất đẳng thức Schur, Vornicu−Schur
và kỹ thuật pqr, một kỹ thuật rất hay để giải các bài toán ba biến.

I. Các kết quả


1. Bất đẳng thức Schur Với mọi a, b, c, r không âm, ta luôn có

ar ( a − b)( a − c) + br (b − c)(b − a) + cr (c − a)(c − b) ≥ 0.

Có khá nhiều cách chứng minh cho bất đẳng thức này, xin được dẫn ra cách đơn giản nhất
Giả sử a ≥ b ≥ c. Ta có

VT = ( a − b)[ ar ( a − c) − br (b − c)] + cr (c − a)(c − b) ≥ 0.

Vậy bất đẳng thức Schur đã được chứng minh.


Hai kết quả được sử dụng nhiều nhất là r = 1 và r = 2. Với r = 1, ta có bất đẳng thức Schur
bậc ba

a( a − b)( a − c) + b(b − c)(b − a) + c(c − a)(c − b) ≥ 0. (1)


1 Tài liệu này được sử dụng vì mục đích học tập. Bất cứ các thao tác, trao đổi nào trên tài liệu này vì mục

đích thương mại đều phải được sự chấp thuận của tác giả.

1
Các dạng tương đương của bất đẳng thức trên là

a3 + b3 + c3 + 3abc ≥ ab( a + b) + bc(b + c) + ca(c + a)

( a + b + c)3 + 9abc ≥ 4( a + b + c)( ab + bc + ca)


abc ≥ ( a + b − c)(b + c − a)(c + a − b)
9abc
a2 + b2 + c2 + ≥ 2( ab + bc + ca)
a+b+c
a b c 4abc
+ + + ≥2
b + c c + a a + b ( a + b)(b + c)(c + a)
(hai bất đẳng thức cuối còn có tên gọi khác là bất đẳng thức Schur bậc ba ở dạng phân thức).
Trường hợp r = 2, chúng ta thu được bất đẳng thức Schur bậc bốn

a2 ( a − b)( a − c) + b2 (b − c)(b − a) + c2 (c − a)(c − b) ≥ 0. (2)

Dạng tương đương của bất đẳng thức trên là

a4 + b4 + c4 + abc( a + b + c) ≥ ab( a2 + b2 ) + bc(b2 + c2 ) + ca(c2 + a2 )

6abc( a + b + c) ≥ (2ab + 2bc + 2ca − a2 − b2 − c2 )( a2 + b2 + c2 + ab + bc + ca).


Bất đẳng thức Schur có hai điểm đẳng thức là a = b = c và a = b, c = 0 cùng các hoán vị
tương ứng. Điều này đã tạo nên độ mạnh cho bất đẳng thức này. Một điều đáng chú ý là
với r chẵn, bất đẳng thức Schur đúng cho mọi số thực a, b, c.
Tiếp theo là kỹ thuật quan trọng mà tác giả muốn chia sẻ với các bạn, đó là kỹ thuật pqr.

2. Kỹ thuật pqr Với mọi số thực a, b, c đặt a + b + c = p, ab + bc + ca = q, abc = r, ta


thu được các đẳng thức sau

a2 + b2 + c2 = p2 − 2q
( a + b)(b + c) + (b + c)(c + a) + (c + a)( a + b) = p2 + q
a3 + b3 + c3 = p3 − 3pq + 3r
ab( a + b) + bc(b + c) + ca(c + a) = pq − 3r
( a + b)(b + c)(c + a) = pq − r
a4 + b4 + c4 = p4 − 4p2 q + 2q2 + 4pr
a2 b2 + b2 c2 + c2 a2 = q2 − 2pr
a3 (b + c) + b3 (c + a) + c3 ( a + b) = p2 q − 2q2 − pr
a3 b3 + b3 c3 + c3 a3 = q3 − 3pqr + 3r2
a4 b4 + b4 c4 + c4 a4 = q4 − 4pq2 r + 2p2 r2 + 4qr2 .

2
Ngoài ra còn rất nhiều đẳng thức khác mà các bạn có thể xây dựng trong quá trình giải
toán.
Từ các đẳng thức trên, ta có một số kết quả (với a, b, c không âm)

p2 ≥ 3q
pq ≥ 9r
p3 ≥ 27r
q2 ≥ 3pr
p3 + 9r ≥ 4pq
p2 q + 3pr ≥ 4q2
p4 + 4q2 + 6pr ≥ 5p2 q.

Từ (1) và (2), ta có các bất đẳng thức sau (với a, b, c không âm)

p(4q − p2 ) (4q − p2 )( p2 − q)
r≥ r≥ .
9 6p

Tuy nhiên 4q − p2 lại có thể âm mà r lại luôn không âm nên ta thường sử dụng các kết quả
sau (rất hiệu quả)

p(4q − p2 ) (4q − p2 )( p2 − q)
   
r ≥ max 0, r ≥ max 0, .
9 6p

Cuối cùng, chúng ta cùng chuyển sang dạng suy rộng của bất đẳng thức Schur, bất đẳng
thức Vornicu−Schur.

3. Bất đẳng thức Vornicu−Schur Với a ≥ b ≥ c là các số thực bất kỳ và x, y, z là các số


thực không âm, xét bất đẳng thức sau

x( a − b)( a − c) + y(b − c)(b − a) + z(c − a)(c − b) ≥ 0.

Bất đẳng thức trên đúng nếu một trong số các điều kiện sau được thỏa mãn

1. x ≥ y (hoặc z ≥ y).

2. ax ≥ by.

3. bz ≥ cy (nếu a, b, c là độ dài ba cạnh của một tam giác).


√ √ √
4. x+ z≥ y.

3
Chứng minh 1. 2. 3. tương tự như cách chứng minh của bất đẳng thức Schur, tuy nhiên
chứng minh cho 4. lại có chút khó khăn.
Viết lại bất đẳng thức dưới dạng
a−c a−c
x· +z· ≥ y,
b−c a−b
hay là
a−b b−c
 
x+z+ x· +z· ≥ y.
b−c a−b
Theo bất đẳng thức AM−GM và tiêu chuẩn 4. ta có

a−b b−c √ √ √
 
x+z+ x· +z· ≥ x + z + 2 xz = ( x + z)2 ≥ y.
b−c a−b

Như vậy 4. đã được chứng minh xong.


Ngoài ra còn rất nhiều các tiêu chuẩn khác mà các bạn có thể tìm thấy trong các tài liệu về
bất đẳng thức.
Chắc đang có nhiều bạn tự hỏi rằng: "Làm cách nào để có thể đưa một bất đẳng thức về
dạng chuẩn tắc theo Vornicu−Schur?" Bằng kinh nghiệm của bản thân, tác giả xin giới
thiệu tới các bạn một cách chuyển đổi, có thể là chưa "tối ưu" và "hoàn hảo" cho lắm. Trước
tiên ta sẽ đưa bất đẳng thức trên về dạng

S a (b − c)2 + Sb (c − a)2 + Sc ( a − b)2 ≥ 0.

Hẳn đây là một việc hết sức đơn giản và dễ dàng đối với phương pháp SOS. Chú ý rằng

∑ Sc (a − b)2 = ∑ Sc (a − b)(a − c + c − b)
= ∑ Sc ( a − b)( a − c) + ∑ Sc (b − c)(b − a)
= ∑ Sc ( a − b)( a − c) + ∑ Sb ( a − b)( a − c) = ∑( Sb + Sc )( a − b)( a − c).

Từ đây, ta tìm được

x = Sb + Sc , y = Sc + S a , z = S a + Sb .

Theo cách phân tích trên, ta có thể nhận ra mối quan hệ mật thiết giữa phương pháp SOS
và bất đẳng thức Vornicu−Schur.
Trong quá trình giải toán, nếu kết hợp kỹ thuật pqr, bất đẳng thức Schur và bất đẳng thức
Vornicu−Schur với các phương pháp như cổ điển, dồn biến, SOS, . . . sẽ tạo ra một sức
mạnh "không tưởng". Điều này sẽ được chứng minh trong phần sau.

4
II. Áp dụng
1. Cho các số thực dương a, b, c. Chứng minh rằng

a2 + b2 + c2 + 2abc + 1 ≥ 2( ab + bc + ca).

(Darij Grinberg)

Lời giải. Theo bất đẳng thức AM−GM, ta có



3 3abc 9abc
2abc + 1 = abc + abc + 1 ≥ 3 a2 b2 c2 = √
3
≥ .
abc a + b+c

Do đó ta chỉ cần chứng minh được

9abc
a2 + b2 + c2 + ≥ 2( ab + bc + ca).
a+b+c

Thế nhưng đây lại là bất đẳng thức Schur bậc ba ở dạng phân thức nên nó hiển nhiên
đúng. Đẳng thức xảy ra khi và chỉ khi a = b = c = 1.

2. Với mọi số thực không âm a, b, c, ta luôn có

2( a2 + b2 + c2 ) + abc + 8 ≥ 5( a + b + c).

(Trần Nam Dũng, Hello IMO 2007)

Lời giải. Sử dụng bất đẳng thức AM−GM, ta có



3
1 abc + abc + 1 3 a2 b2 c2 3abc 9abc
abc + = ≥ = √3
≥ ,
2 2 2 2 abc 2 ( a + b + c)


5 5h 2 i
5( a + b + c) = · 2 · 3 · ( a + b + c) ≤ 3 + ( a + b + c)2 .
6 6
Do đó, ta chỉ cần chứng minh được
  9abc 15 5h i
2 a2 + b2 + c2 + + ≥ 9 + ( a + b + c)2 ,
2( a + b + c) 2 6

hay là
  27abc
7 a2 + b2 + c2 + ≥ 10( ab + bc + ca).
( a + b + c)

5
Mà 4 a2 + b2 + c2 ≥ 4( ab + bc + ca) (theo AM−GM) nên bài toán được đưa về


9abc
a2 + b2 + c2 + ≥ 2( ab + bc + ca).
a+b+c

Nhưng đây chỉ là bất đẳng thức Schur bậc ba ở dạng phân thức. Đẳng thức xảy ra khi và
chỉ khi a = b = c = 1.

3. Nếu a, b, c là các số thực không âm thì

( a2 + 2)(b2 + 2)(c2 + 2) ≥ 9( ab + bc + ca).

(APMO 2004)

Lời giải. Ta có, theo bất đẳng thức AM−GM và bất đẳng thức Schur thì

( a2 + 2)(b2 + 2)(c2 + 2)
= a2 b2 c2 + 1 + 1 + 2( a2 b2 + 1) + 2(b2 c2 + 1) + 2(c2 a2 + 1) + 4( a2 + b2 + c2 )
√3
≥ 3 a2 b2 c2 + 4( ab + bc + ca) + 4( a2 + b2 + c2 )
9abc
≥ + 4( ab + bc + ca) + 4( a2 + b2 + c2 )
a+b+c
≥ 2( ab + bc + ca) − ( a2 + b2 + c2 ) + 4( ab + bc + ca) + 4( a2 + b2 + c2 )
≥ 9( ab + bc + ca).

Bất đẳng thức được chứng minh xong. Đẳng thức xảy ra khi và chỉ khi a = b = c = 1.

4. Giả sử a, b, c là các số thực không âm thỏa mãn a5 + b5 + c5 = 3. Chứng minh

a6 b6 + b6 c6 + c6 a6 ≤ 3.

(Vasile Cirtoaje, Gabriel Dospinescu)

Lời giải. Sử dụng bất đẳng thức AM−GM,


 
5a6 b6 ≤ a5 b5 a5 + b5 + 3 .

Bằng cách thiết lập hai bất đẳng thức tương tự cho các số hạng còn lại rồi cộng vào, ta sẽ
chỉ ra rằng
xy( x + y + 3) + yz( y + z + 3) + zx( z + x + 3) ≤ 15,

6
trong đó x = a5 , y = b5 , c = z5 và x + y + z = 3. Bất đẳng thức trên tương đương với
( x + y + z)( xy + yz + zx) + 3( xy + yz + zx) ≤ 15 + 3xyz,
hay là
2( xy + yz + zx) ≤ 5 + xyz.
Sử dụng bất đẳng thức Schur bậc ba, ta có
( x + y + z) 4( xy + yz + zx) − ( x + y + z)2
 
5 + xyz ≥ 5 +
9
4( xy + yz + zx) − 9 6 + 4( xy + yz + zx)
= 5+ = .
3 3
Do đó ta chỉ cần chứng minh được
6 + 4( xy + yz + zx)
2( xy + yz + zx) ≤ ,
3
tương đương với xy + yz + zx ≤ 3, hiển nhiên đúng theo AM−GM. Đẳng thức xảy ra khi
và chỉ khi a = b = c = 1.

5. Với các số thực không âm a, b, c thỏa mãn ab + bc + ca > 0, bất đẳng thức sau luôn đúng
a2 + bc b2 + ca c2 + ab
+ + ≥ 3.
b2 − bc + c2 c2 − ca + a2 a2 − ab + b2
(Ngô Đức Lộc)

Lời giải. Theo bất đẳng thức AM−GM thì


2 2
a2 + bc a2 + bc 4 a2 + bc
= 2 ≥ 2 .
b2 − bc + c2 (b − bc + c2 )( a2 + bc) ( a + b2 + c2 )2
Làm tương tự cho các số hạng còn lại rồi cộng vào, ta cần chứng minh
4( a2 + bc)2 + 4(b2 + ca)2 + 4(c2 + ab)2 ≥ 3( a2 + b2 + c2 )2 .
Bất đẳng thức này tương đương với
a4 + b4 + c4 + 8abc( a + b + c) ≥ 2( a2 b2 + b2 c2 + c2 a2 ).
Sử dụng bất đẳng thức Schur bậc bốn và bất đẳng thức AM−GM, ta có
a4 + b4 + c4 + 8abc( a + b + c) ≥ a4 + b4 + c4 + abc( a + b + c)
≥ ab( a2 + b2 ) + bc(b2 + c2 ) + ca(c2 + a2 )
≥ 2( a2 b2 + b2 c2 + c2 a2 ).

7
Đẳng thức xảy ra khi và chỉ khi a = b, c = 0 cùng các hoán vị.

6. Cho a, b, c là các số thực không âm, tất cả không đồng thời bằng 0. Chứng minh

3 a3 + b3 + c3

p p p
2 2 2
≥ a2 − ab + b2 + b2 − bc + c2 + c2 − ca + a2 .
a +b +c

Lời giải. Sử dụng bất đẳng thức Cauchy−Schwarz, ta có


   2
a3 + b3 + c3 ( a + b + c) ≥ a2 + b2 + c2 ,

suy ra s
a3 + b3 + c3 a3 + b3 + c3
≥ .
a2 + b2 + c2 a+b+c
Mặt khác, cũng từ bất đẳng thức này thì
p q q
a 2 − ab + b2 ≤ 3 ( a 2 − ab + b2 ) = 3 [2 ( a2 + b2 + c2 ) − ( ab + bc + ca)].
∑ ∑
Do đó ta chỉ cần chứng minh được

3 a3 + b3 + c3
  
≥ 2 a2 + b2 + c2 − ( ab + bc + ca),
a+b+c
hay là
a3 + b3 + c3 + 3abc ≥ ab( a + b) + bc(b + c) + ca(c + a).
Tuy nhiên đây lại là bất đẳng thức Schur bậc ba nên nó hiển nhiên đúng. Đẳng thức xảy ra
khi và chỉ khi a = b = c hoặc ( a, b, c) ∼ (1, 1, 0) cùng các hoán vị.

7. Cho a, b, c là các số thực không âm trong chúng không có hai số nào đồng thời bằng 0. Chứng
minh rằng

a3 b3 c3
√ +√ +√ ≥ a2 + b2 + c2 .
b2 − bc + c2 c2 − ca + c2 a2 − ab + b2
(Võ Quốc Bá Cẩn)

Lời giải. Sử dụng bất đẳng thức Cauchy−Schwarz, ta có


2
a3 a2 + b2 + c2
∑ √b2 − bc + c2 ≥ ∑ a√b2 − bc + c2 .

8
Do đó ta chỉ cần chứng minh được
p p p
a b2 − bc + c2 + b c2 − ca + a2 + c a2 − ab + b2 ≤ a2 + b2 + c2 .

Lại sử dụng bất đẳng thức Cauchy−Schwarz,


 p 2 h  i
∑a b2 − bc + c2 ≤ ∑a ∑a b2 − bc + c2 .

Như vậy, ta phải chứng minh


 2 h  i
a2 + b2 + c2 ≥ ∑a ∑a b2 − bc + c2 .

Bất đẳng thức này tương đương với

a4 + b4 + c4 + abc( a + b + c) ≥ a3 (b + c) + b3 (c + a) + c3 ( a + b),

là bất đẳng thức Schur bậc bốn. Đẳng thức xảy ra khi và chỉ khi a = b = c hoặc a = b, c = 0
cùng các hoán vị.

8. Chứng minh bất đẳng thức sau với các số thực không âm a, b, c thỏa mãn ab + bc + ca > 0

a(b + c) b( a + c) c( a + b) 3[( a − b)(b − c)(c − a)]2


+ + ≥ 2 + .
b2 + bc + c2 c2 + ca + a2 a2 + ab + b2 ( a2 + ab + b2 )(b2 + bc + c2 )(c2 + ca + a2 )

(Nguyễn Huy Tùng)

Lời giải. Ta có
 
a(b + c) 1 a(b + c)( ab + bc + ca)
∑ b2 + bc + c2 − 2 = ab + bc + ca ∑ b2 + bc + c2
− a(b + c)

1 a(b + c) [b( a − b) − c(c − a)]


=
ab + bc + ca ∑ b2 + bc + c2
ab( a − b)(b + c) ca(c − a)(b + c)
 
1
= −∑
ab + bc + ca ∑ b2 + bc + c2 b2 + bc + c2
ab( a − b)(b + c) ab( a − b)(c + a)
 
1
= −∑
ab + bc + ca ∑ b2 + bc + c2 c2 + ca + a2
1 ab( a − b)2 ( ab + bc + ca)
=
ab + bc + ca ∑ (b2 + bc + c2 )(c2 + ca + a2 )
ab( a − b)2
=∑ 2 .
(b + bc + c2 )(c2 + ca + a2 )

9
Nên bất đẳng thức đã cho có thể được viết lại thành

ab( a − b)2 3 [( a − b)(b − c)(c − a)]2



∑ (b2 + bc + c2 )(c2 + ca + a2 ) (a2 + ab + b2 )(b2 + bc + c2 )(c2 + ca + a2 ) ,
hay là
∑ ab(a2 + ab + b2 )(a − b)2 ≥ 3 [(a − b)(b − c)(c − a)]2 .
Sau khi sử dụng bất đẳng thức AM−GM, ta sẽ chứng minh bất đẳng thức mạnh hơn là

a2 b2 ( a − b)2 + b2 c2 (b − c)2 + c2 a2 (c − a)2 ≥ ( a − b)2 (b − c)2 (c − a)2 ,

tương đương với

2abc[ a( a − b)( a − c) + b(b − c)(b − a) + c(c − a)(c − b)] ≥ 0,

hiển nhiên đúng theo bất đẳng thức Schur. Đẳng thức xảy ra khi và chỉ khi a = b = c hoặc
a = b, c = 0 cùng các hoán vị.
Nhận xét. Bất đẳng thức trên là một ví dụ điển hình cho sự kết hợp giữa của phương pháp
phân tích bình phương SOS và bất đẳng thức Schur.

9. Xác định hằng số a nhỏ nhất sao cho bất đẳng thức sau luôn được thỏa mãn với mọi x, y, z
không âm
 3− a
x + y + z a xy + yz + zx 2
  
( x + y)( y + z)( z + x)
≥ .
3 3 8
(Iurie Borieco, Ivan Borsenco)

3 ln 3 − 4 ln 2
Lời giải. Cho x = y = 1, z = 0, ta thu được a ≥ a0 = ≈ 1.81884 . . . . Ta
2 ln 2 − ln 3
sẽ chứng minh đây là giá trị nhỏ nhất của a, tức là
  a0   3−2a0
x+y+z xy + yz + zx ( x + y)( y + z)( z + x)
≥ .
3 3 8

Bất đẳng thức trên là một bất đẳng thức đồng bậc cho x, y, z nên không mất tính tổng quát,
ta có thể chuẩn hóa cho p = x + y + z = 1. Đặt q = xy + yz + zx, r = xyz, khi đó bất đẳng
thức trên được viết lại thành
3− a0
8q 2
r + 3+a0 ≥ q.
3 2

10
Xét hai trường hợp
Trường hợp 1. 1 ≥ 4q ≥ 0. Ta có
3− a0 3− a0
8q 2 8q 2
r+ 3+ a0 −q ≥ 3+ a0 −q
3 2 3 2
 
    a02−1
3− a0 8 a0 −1 3− a0
 38+a 1
=q 2
3+ a0 −q 2 ≥q 2 −  = 0.
3 2 3 2
0 4

1 1 4q − 1
Trường hợp 2. ≥ q ≥ . Áp dụng bất đẳng thức Schur bậc ba, ta có r ≥ . Điều
3 4 9
này dẫn đến
3− a0 3− a0 3− a0
8q 24q − 1 8q 2 8q 2 5q + 1
r + 3+ a0 − q ≥ + 3+ a0 − q = 3+ a0 − = f (q).
3 2 9 3 2 3 2 9
 
1 1
Dễ dàng kiểm tra được f (q) là hàm lõm với mọi q ∈ , . Do đó
4 3
    
1 1
f (q) ≥ min f ,f .
3 4
   
1 1
Mà f = f = 0 nên f (q) ≥ 0. Điều này chứng tỏ trong mọi trường hợp, ta luôn
3 4
có 3− a0
8q 2
r + 3+a0 ≥ q.
3 2
Vì vậy,
3 ln 3 − 4 ln 2
min a = .
2 ln 2 − ln 3

10. Cho a, b, c là các số thực không âm sao cho trong chúng cho ít nhất hai số dương thỏa mãn
a2 + b2 + c2 = 1. Chứng minh
a3 b3 c3 √
2 2
+ 2 2
+ 2 2
≥ 2.
b − bc + c c − ca + a a − ab + b
(Võ Quốc Bá Cẩn)

Lời giải. Sử dụng bất đẳng thức Cauchy−Schwarz, ta có


a3 a4 ( a2 + b2 + c2 )2 1
∑ b2 − bc + c2 =∑ 2 2
≥ 2 2
= .
a(b − bc + c ) ∑ a(b − bc + c ) ∑ ab( a + b) − 6abc

11
Do đó ta chỉ cần chứng minh được
√  
2 ∑ ab( a + b) − 6abc ≤ 1.

p2 − 1
Đổi biến theo p, q, r, ta có q = . Bất đẳng thức trên tương đương với
2

2 + 18r − p( p2 − 1) ≥ 0.

Nếu p ≤ 2 thì bất đẳng thức trên là hiển nhiên. Xét trường hợp ngược lại, khi đó theo
p(4q − p2 ) p( p2 − 2)
bất đẳng thức Schur, ta có r ≥ = . Suy ra
9 9
√ √  √  √ 
2 + 18r − p( p2 − 1) ≥ 2 + 2p( p2 − 2) − p( p2 − 1) = p − 2 p2 + p 2 − 1 ≥ 0.

1
Bài toán được chứng minh. Đẳng thức xảy ra khi và chỉ khi a = b = √ , c = 0 cùng các
2
hoán vị.

11. Cho a, b, c là các số thực không âm có tổng bằng 1. Chứng minh

1
a4 (b + c) + b4 (c + a) + c4 ( a + b) ≤ .
12
(Vasile Cirtoaje)

Lời giải. Đổi biến theo p, q, r, bất đẳng thức trên được viết lại thành

1
q(1 − 3q) + r(5q − 1) ≤ .
12
Xét hai trường hợp
1
Trường hợp 1. q ≤ . Ta có
5
1 1 1
q(1 − 3q) + r(5q − 1) ≤ q(1 − 3q) = · 4 · 3q · (1 − 3q) ≤ · (1 − 3q + 3q)2 = .
12 12 12
1
Trường hợp 2. q > . Ta có
5
q(5q − 1) 1 1 1
q(1 − 3q) + r(5q − 1) ≤ q(1 − 3q) + = (32q − 88q2 − 3) + < .
9 36 12 12

12
√ √
3+ 3 3− 3
Phép chứng minh hoàn tất. Đẳng thức xảy ra khi và chỉ khi a = ,b = ,c = 0
6 6
cùng các hoán vị.

12. Cho các số thực không âm a, b, c thỏa mãn điều kiện ab + bc + ca + 6abc = 9. Chứng minh
rằng bất đẳng thức sau luôn đúng

a + b + c + 3abc ≥ 6.

(Lê Trung Kiên, Võ Quốc Bá Cẩn)

Lời giải. Đổi biến theo p, q, r, ta có q + 6r = 9, và bất đẳng thức đã cho tương đương
với
p + 3r ≥ 6,
hay là
2p − q ≥ 3.
Nếu p ≥ 6 thì bất đẳng thức trên là hiển nhiên. Xét 6 ≥ p ≥ 3, áp dụng bất đẳng thức
p(4q − p2 )
Schur bậc ba, ta có r ≥ . Thế thì
9
27 = 3q + 18r ≥ 3q + 2p(4q − p2 ).

Suy ra
2p3 + 27
q≤
8p + 3
Vì vậy
2p3 + 27 ( p + 1)( p − 3)(6 − p)
2p − q − 3 ≥ 2p − −3 = ≥ 0.
8p + 3 8p + 3
Vậy bất đẳng thức được chứng minh xong. Đẳng thức xảy ra khi và chỉ khi a = b = 3, c = 0
cùng các hoán vị.
Nhận xét. k = 3 cũng là hằng số tốt nhất để bất đẳng thức sau đúng (với cùng điều kiện)

a + b + c + kabc ≥ 3 + k.

13. Cho các số thực không âm a, b, c có tổng bằng 3. Chứng minh


1 1 1 3
+ + ≤ .
6 − ab 6 − bc 6 − ca 5
(Vasile Cirtoaje)

13
Lời giải. Đặt theo p, q, r và biến đổi tương đương, rút gọn, bất đẳng thức đã cho được
viết lại thành
108 − 48q + 39r − 3r2 ≥ 0,
hay là
4(9 − 4q + 3r) + r(1 − r) ≥ 0.
Bất đẳng thức trên hiển nhiên đúng do 9 − 4q + 3r ≥ 0 (bất đẳng thức Schur bậc ba)
và 1 ≥ r (AM−GM) nên ta có điều cần chứng minh. Đẳng thức xảy ra khi và chỉ khi
a = b = c = 1 hoặc a = b = 23 , c = 0 cùng các hoán vị.

14. Cho các số thực không âm a, b, c trong chúng không có hai số nào đồng thời bằng 0 và k ≥ 3.
Chứng minh rằng

1 1 1 k 2 k+1
+ + + ≥√ .
a+b b+c c+a a+b+c ab + bc + ca
(Phạm Kim Hùng)

Lời giải. Đổi biến theo p, q, r và chuẩn hóa cho p = 1. Bất đẳng thức đã cho tương đương
với √
1+q 2 k+1
+k ≥ √ .
q−r q
Ta có √
1+q 1+q 1 2 k+1
+k ≥ +k = +k+1 ≥ √ .
q−r q q q
√ !
k − 1 + k2 − 2k − 3
Đẳng thức xảy ra khi và chỉ khi ( a, b, c) = x, x, 0 cùng các hoán vị.
2
15. Với các số thực không âm a, b, c trong chúng không có hai số nào cùng bằng 0, ta luôn có
1 1 1 9
2
+ 2
+ 2
≥ .
( a + b) (b + c) (c + a) 4( ab + bc + ca)
(Ji Chen, Iran TST 1996, Crux Mathematicorum)

Lời giải. Đổi biến theo p, q, r, bất đẳng thức cần chứng minh tương đương với

( p2 + q)2 − 4p( pq − r) 9
≥ ,
( pq − r)2 4q
hay là
3pq( p3 + 9r − 4pq) + q( p4 + 6q2 + 4pr − 5p2 q) + r( pq − 9r) ≥ 0.

14
Bất đẳng thức trên đúng do p3 + 9r ≥ 4pq, p4 + 6q2 + 4pr ≥ 5p2 q (Schur) và pq ≥ 9r
(AM−GM). Đẳng thức xảy ra khi và chỉ khi a = b = c hoặc a = b, c = 0 cùng các hoán vị.

16. Cho các số thực không âm a, b, c trong chúng có ít nhất hai số dương. Chứng minh
2  2  2
5 a2 + b2 + c2

a b c 1
+ + + ≥ · .
b+c c+a a+b 2 4 ab + bc + ca
(Trần Quốc Anh, Dương Đức Lâm)

Lời giải. Nhân cả hai vế của bất đẳng thức đã cho với ( a + b)(b + c)(c + a), ta có thể
viết lại nó dưới dạng
a2 ( a + b)(c + a) ( a + b)(b + c)(c + a) 5 ( a2 + b2 + c2 )( a + b)(b + c)(c + a)
∑ + ≥ · ,
b+c 2 4 ab + bc + ca
hay là
a2 ( a − b)( a − c) ∑ ab( a + b)
∑ + 2 ∑ a3 + abc +
b+c 2
5   5 abc( a + b + c2 )
2 2
a2 ∑ a − ·

≥ ∑ .
4 4 ab + bc + ca
a2 ( a − b)( a − c)
Theo bất đẳng thức Vornicu−Schur thì ∑ ≥ 0 và theo bất đẳng thức
b+c
a2 + b2 + c2
AM−GM thì ≥ 1. Do đó ta chỉ cần chứng minh được
ab + bc + ca
∑ ab( a + b) 5  5abc

2 ∑ a3 + abc + ≥ ∑ a 2
∑ a − ,
2 4 4
tương đương với
a3 + b3 + c3 + 3abc ≥ ab( a + b) + bc(b + c) + ca(c + a),
là bất đẳng thức Schur bậc ba. Phép chứng minh được hoàn tất.

17. Với mọi số dương x, y, z, ta luôn có


1 1 1 1
+ 2 + 2 ≤ p .
2x2 + yz 2y + zx 2z + xy 3
x y2 z2
2

Lời giải. Đặt a = xy, b = yz, c = zx, bất đẳng thức trên trở thành
a b c 1
+ 2 + 2 ≤ √ ,
a2 + 2bc b + 2ca c + 2ab 3
abc

15
tương đương với
a( a − b)( a − c) 1 a+b+c
∑ 2
+√ 3
≥ .
a + 2bc abc ab + bc + ca
a( a − b)( a − c) √
Theo bất đẳng thức Vornicu−Schur, ta có ∑ 2
≥ 0. Do đó, nếu 3 abc ≤
a + 2bc
ab + bc + ca
thì bất đẳng thức trên là hiển nhiên. Trong trường hợp ngược lại, ta có
a+b+c
√3
1 a2 b2 c2 ( ab + bc + ca)2
√ = ≥ .
3
abc abc abc( a + b + c)2

Do đó ta chỉ cần chứng minh được

a b c ( ab + bc + ca)2
+ + ≤ .
a2 + 2bc b2 + 2ca c2 + 2ab abc( a + b + c)2

Đổi biến theo p, q, r và chuẩn hóa cho p = 1, bất đẳng thức trên được viết lại thành

q(2q − 9r) q2
≤ ,
27r2 + 2(2 − 9q)r + 2q3 r
hay là
f (r) = 9(1 + 3q)r2 + 2q(1 − 9q)r + 2q4 ≥ 0.
Nếu 1 ≥ 9q thì bất đẳng thức hiển nhiên đúng. Nếu 9q ≥ 1 thì

∆0f = q2 (1 − 9q)2 − 18q4 (1 + 3q) = q2 (1 − 3q)[6q(3q − 1) − (9q − 1)] ≤ 0.

Do đó, ta luôn có f (r) ≥ 0. Phép chứng minh được hoàn tất. Đẳng thức xảy ra khi và chỉ
khi x = y = z = 1.

18. Cho a, b, c là các số thực dương. Chứng minh

a2 + bc b2 + ca c2 + ab
+ + ≥ 2.
b2 + bc + c2 c2 + ca + a2 a2 + ab + b2
(Vasile Cirtoaje)

Lời giải. Ta có

a2 + bc a2 + bc a(b + c) a(b + c)
∑ b2 + bc + c2 − 2 = ∑ b2 + bc + c2 − ∑ b2 + bc + c2 + ∑ b2 + bc + c2 − 2
( a − b)( a − c) ab( a − b)2
=∑ + ∑ .
b2 + bc + c2 (b2 + bc + c2 )(c2 + ca + a2 )

16
Dễ thấy
( a − b)( a − c)
∑ ≥0 (Vornicu−Schur)
b2 + bc + c2

ab( a − b)2
∑ (b2 + bc + c2 )(c2 + ca + a2 ) ≥ 0
nên bất đẳng thức trên đúng. Đẳng thức xảy ra khi và chỉ khi a = b = c.

19. Cho a, b, c là các số thực không âm, trong chúng không có hai số nào cùng bằng 0. Chứng
minh
a3 (b + c) b3 (c + a) c3 ( a + b)
a2 + b2 + c2 ≥ 2 + 2 + 2 ≥ ab + bc + ca.
a + bc b + ca c + ab
(Phạm Hữu Đức)

Lời giải. Trước hết ta sẽ chứng minh bất đẳng thức bên trái. Ta có

a3 (b + c) a2 ( a − b)( a − c)
 
a(b + c)
∑ a2 − ∑ a2 + bc
= ∑ a 2
1 −
a2 + bc
= ∑ a2 + bc
.

Không mất tính tổng quát giả sử a ≥ b ≥ c. Khi đó

a2 b2 c( a3 − b3 )
− = ≥0
a2 + bc b2 + ca ( a2 + bc)(b2 + ca)

nên theo định lý 1. ta có điều phải chứng minh. Với bất đẳng thức bên phải, dễ thấy bất
đẳng thức trên là hiển nhiên khi abc = 0 nên ta chỉ cần xét abc > 0. Ta có

2a3 (b + c) a(b + c)( a2 − bc)


∑ − ∑ a(b + c) = ∑
a2 + bc a2 + bc
a(b + c)( a2 − bc)
 
=∑ − ( a − bc) + ∑( a2 − bc)
2
a2 + bc
( a2 − bc)( a − b)( a − c)
= ∑( a − b)( a − c) − ∑
a2 + bc
( a − b)( a − c)
= 2abc ∑ .
a3 + abc
Mặt khác, do a ≥ b ≥ c nên

1 1 b3 − c3
− = ≥ 0,
c3 + abc b3 + abc (b3 + abc)(c3 + abc)

17
nên cũng theo định lý 1. ta có điều phải chứng minh. Bất đẳng thức bên trái có đẳng thức
khi a = b = c hoặc a = b, c = 0 cùng các hoán vị, bất đẳng thức bên phải có đẳng thức khi
a = b = c.

20. Cho các số thực không âm a, b, c trong chúng có ít nhất hai số dương. Chứng minh bất đẳng
thức
a2 + 2bc b2 + 2ca c2 + 2ab 3( a + b + c)
+ + ≥ .
b+c c+a a+b 2

Lời giải. Nhân cả hai vế của bất đẳng thức đã cho với a + b + c và chú ý rằng

( a2 + 2bc)( a + b + c) a( a2 + 2bc)
= + a2 + 2bc,
b+c b+c

nên nó có thể được viết lại thành

a( a2 + 2bc) b(b2 + 2ca) c(c2 + 2ab) ( a + b + c)2


+ + ≥ ,
b+c c+a a+b 2
tương đương với

a( a − b)( a − c)
 
 a 1 2
∑ + ∑ ab ∑ b+c ≥ ∑a .
b+c 2

Sử dụng bất đẳng thức Cauchy−Schwarz, ta có

( a + b + c)2 ( a + b + c)2
 
a b c
( ab + bc + ca) + + ≥ ( ab + bc + ca) · = .
b+c c+a a+b 2( ab + bc + ca) 2

Cộng việc còn lại ta sẽ chứng minh

a( a − b)( a − c) b(b − a)(b − c) c(c − a)(c − b)


+ + ≥ 0.
b+c c+a a+b

Không mất tính tổng quát giả sử a ≥ b ≥ c. Dễ thấy b+a c ≥ c+b a nên định lý 1. cho ta điều
phải chứng minh. Đẳng thức xảy ra khi và chỉ khi a = b = c hoặc a = b, c = 0 cùng các
hoán vị.

21. Chứng minh với các số thực không âm a, b, c thỏa mãn ab + bc + ca > 0, ta có bất đẳng
thức
a2 b2 c2 1
+ + ≤ .
(2a + b)(2a + c) (2b + c)(2b + a) (2c + a)(2c + b) 3

18
Lời giải. Ta có

a2 a2
 
1 a
−∑ = −
∑ 3(a + b + c) (2a + b)(2a + c)
3 (2a + b)(2a + c)
a[(2a + b)(2a + c) − 3a( a + b + c)]
=∑
3( a + b + c)(2a + b)(2a + c)
1 a( a − b)( a − c)
= · .
3( a + b + c) ∑ (2a + b)(2a + c)
Giả sử a ≥ b ≥ c. Khi đó
a b a b
≥ , ≥ .
2a + b 2b + a 2a + c 2b + c
Suy ra
a2 b2
≥ .
(2a + b)(2a + c) (2b + c)(2b + a)
Từ đó, theo định lý 2. ta có điều cần chứng minh. Đẳng thức xảy ra khi và chỉ khi a = b = c
hoặc a = b, c = 0 cùng các hoán vị.

22. Với các số dương a, b, c ta luôn có


s
a b c 9( a3 + b3 + c3 )
+ + ≥33 .
b c a ( a + b + c)3

(Nguyễn Huy Tùng)

Lời giải. Sử dụng bất đẳng thức quen thuộc 4( x + y + z)3 ≥ 27( x2 y + y2 z + z2 x + xyz)
với x = ba , y = bc , z = ac , ta được
3
a2 b2 c2 a3 + b3 + c3
    
a b c 27 27
+ + ≥ + + +1 = +1 .
b c a 4 bc ca ab 4 abc

Do đó ta chỉ cần chứng minh được

a3 + b3 + c3 36( a3 + b3 + c3 )
+1 ≥ .
abc ( a + b + c)3

Sử dụng các hằng đẳng thức

a3 + b3 + c3 ( a + b + c) ∑( a − b)( a − c)
+1−4 =
abc abc

19

36( a3 + b3 + c3 ) 4 ∑(8a + 5b + 5c)( a − b)( a − c)
3
−4 = ,
( a + b + c) ( a + b + c)3
ta có thể viết lại bất đẳng thức trên dưới dạng

x( a − b)( a − c) + y(b − c)(b − a) + z(c − a)(c − b) ≥ 0,

với
a + b + c 4(8a + 5b + 5c)
x= −
abc ( a + b + c)3
và các biểu thức y, z tương tự. Không mất tính tổng quát giả sử a ≥ b ≥ c. Dễ thấy
x ≤ y ≤ z nên ta chỉ cần chứng minh x ≥ 0 là đủ. Chuẩn hóa cho b + c = 1, khi đó a ≥ 12

a + 1 4(8a + 5) 4( a + 1) 4(8a + 5)
x= − ≥ −
abc ( a + 1)3 a ( a + 1)3
4( a + 4a − 2a2 − a + 1)
4 3
 
1
= ≥0 a≥ .
a( a + 1)3 2
Vậy ta luôn có z ≥ y ≥ x ≥ 0. Từ đây, theo định lý 1. ta có điều phải chứng minh. Đẳng
thức xảy ra khi và chỉ khi a = b = c.

Qua những bài toán trên hẳn các bạn đã thấy được tầm ứng dạng rộng rãi và hiệu quả
của bất đẳng thức Schur, Vornicu−Schur và kỹ thuật pqr. Sau đây là một số bài tập để các
bạn thử sức

Bài tập 1. Với a, b, c là các số thực không âm thỏa mãn điều kiện a2 + b2 + c2 = 3, hãy
chứng minh bất đẳng thức

12 + 9abc ≥ 7( ab + bc + ca).

(Vasile Cirtoaje)

Bài tập 2. Chứng minh rằng

a(b + c) b(c + a) c( a + b) 2( a − b)2 (b − c)2 (c − a)2


2
+ 2 + 2 ≤ 2+ 2
a + 2bc b + 2ca c + 2ab ( a + 2bc)(b2 + 2ca)(c2 + 2ab)

trong đó a, b, c là các số thực không âm thỏa mãn ab + bc + ca > 0.


(Lê Trung Kiên)

20
Bài tập 3. Cho a, b, c là các số thực không âm, trong chúng không có hai số nào cùng
bằng 0. Chứng minh
s s s √
3
2 + bc 2 + ca 2
3 a 3 b 3 c + ab 9 abc
+ + ≥ .
b2 + c2 c2 + a2 a2 + b2 a+b+c

(Phạm Hữu Đức)

Bài tập 4. Cho a, b, c là các số thực không âm, trong chúng không có hai số nào cùng
bằng 0. Chứng minh
s s s
a2 b2 c2
2 2
+ 2 2
+ ≥ 1.
4b + bc + 4c 4c + ca + 4a 4a + ab + 4b2
2

(Phạm Kim Hùng, Võ Quốc Bá Cẩn)

Bài tập 5. Chứng minh với mọi số dương a, b, c,

a2 − bc b2 − ca c2 − ab
√ +√ +√ ≥ 0.
a2 + 2b2 + 3c2 b2 + 2c2 + 3a2 c2 + 2a2 + 3b2

Bài tập 6. Với các số thực a, b, c thỏa mãn a2 + b2 + c2 = 9, ta luôn có

2( a + b + c) − abc ≤ 10.

(Trần Nam Dũng, Vietnamese MO 2002)

Bài tập 7. Cho các số dương a, b, c có tổng bằng 3. Tìm hằng số k tốt nhất để bất đẳng
thức sau luôn đúng
ab bc ca
+ + + kabc ≥ 3 + k.
c a b
(Võ Quốc Bá Cẩn)

Bài tập 8. Cho a, b, c là các số thực không âm, trong chúng không có hai số nào đồng
thời bằng 0. Chứng minh

a b c 18
+ 3 + 3 ≥ .
b3 +c 3 c +a 3 a +b 3 2 2 2
5( a + b + c ) − ( ab + bc + ca)

(Michael Rozenberg)

21
Bài tập 9. Cho a, b, c là các số thực không âm thỏa mãn ab + bc + ca + abc = 4. Chứng
minh
a3 + b3 + c3 + 9abc ≥ 4( a + b + c).
(Võ Quốc Bá Cẩn, Trần Quốc Anh)

Bài tập 10. Cho a, b, c là các số dương có tổng bình phương bằng 3. Chứng minh
√ √ √
a2 b + c b2 c + a c2 a + b
√ +√ +√ ≤ 3.
a2 + bc b2 + ca c2 + ab
(Phạm Hữu Đức)

Bài tập 11. Cho các số dương a, b, c. Chứng minh bất đẳng thức
s s s
a3 + abc b3 + abc c3 + abc 3
3
+ 3
+ 3

(b + c) (c + a) ( a + b) 2

(Nguyễn Văn Thạch)

Bài tập 12. Với a, b, c là các số dương có tổng bằng 3, hãy chứng minh

1 1 1
2
+ 2 + 2 ≥ a2 + b2 + c2
a b c
(Romania TST 2005)

Cảm ơn các bạn đã theo dõi bài viết này!

22

You might also like